The Stacks project

Remark 10.66.12. Let $\varphi : R \to S$ be a ring map. Let $M$ be an $S$-module. Denote $f : \mathop{\mathrm{Spec}}(S) \to \mathop{\mathrm{Spec}}(R)$ the associated map on spectra. Then we have

\[ f(\text{Ass}_ S(M)) \subset \text{Ass}_ R(M) \subset \text{WeakAss}_ R(M) \subset f(\text{WeakAss}_ S(M)) \]

see Lemmas 10.63.11, 10.66.11, and 10.66.6. In general all of the inclusions may be strict, see Remarks 10.63.12 and 10.66.10. If $S$ is Noetherian, then all the inclusions are equalities as the outer two are equal by Lemma 10.66.9.


Comments (0)

There are also:

  • 3 comment(s) on Section 10.66: Weakly associated primes

Post a comment

Your email address will not be published. Required fields are marked.

In your comment you can use Markdown and LaTeX style mathematics (enclose it like $\pi$). A preview option is available if you wish to see how it works out (just click on the eye in the toolbar).

Unfortunately JavaScript is disabled in your browser, so the comment preview function will not work.

All contributions are licensed under the GNU Free Documentation License.




In order to prevent bots from posting comments, we would like you to prove that you are human. You can do this by filling in the name of the current tag in the following input field. As a reminder, this is tag 05C7. Beware of the difference between the letter 'O' and the digit '0'.